Understanding Gauss's Law: Where Does the Argument Break Down?

In summary: Interesting question. Upon reflection, my guess is that the integral form of Gauss's Law rests on an unstated assumption that the average charge density outside of the closed surface is zero. In practice, that will usually be close enough to correct to ignore any inaccuracy. A re-stated Gauss's Law that coped with this thought experiment might replace Q, the charge inside the surface, by ##(Q-V\rho_0)## where ##V## is the volume inside the surface and ##\rho_0## is the average charge density across all space. Similarly, for the differential case, one could replace ##\rho## by ##\rho-\rho_0##.
  • #36
tom.stoer said:
The solution can be much more complex b/c you can add any ##\vec{E}_0## with ##\nabla\vec{E}_0 = 0##, e.g.

$$\vec{E}_0 = (0,y\,f(x),-z\,f(x))$$
You also need to satisfy ##\nabla \times \vec E = 0## and with that ##\vec E_0## generally
$$
\nabla \times \vec E = f'(x) [z \vec e_y + y \vec e_z] \neq 0.
$$

Edit: I will give you that you can add any divergence and curl free field.
 
Last edited:
Physics news on Phys.org
  • #37
Orodruin said:
You also need to satisfy ##\nabla \times \vec E = 0## and with that ##\vec E_0## generally
$$
\nabla \times \vec E = f'(x) [z \vec e_y + y \vec e_z] \neq 0.
$$
you are right; sorry for being imprecise

Orodruin said:
Edit: I will give you that you can add any divergence and curl free field.
yes, that's basically the result; we may also discuss this in terms of the potential and harmonic functions
 
  • #38
tom.stoer said:
yes, that's basically the result; we may also discuss this in terms of the potential and harmonic functions
Indeed, in fact this is how I would prefer to approach this. Start with the particular solution ##\vec E = \vec x \rho/3\epsilon_0## and then solve ##\nabla^2 V = 0##.

The general solution to the homogeneous equation for the potential is on the form
$$
V = \sum_{\ell = 0}^\infty \sum_{|m| \leq \ell} f_{\ell m} Y_{\ell m}(\theta,\varphi) r^\ell
$$
in spherical coordinates, where ##f_{\ell m}## must be chosen to satisfy whatever behaviour is imposed at infinity.

Edit: The ##\ell = 0## term is a constant that does not change the field at all. The ##\ell = 1## terms essentially correspond to the addition of a constant field. The higher ##\ell## terms all correspond to fields that are not constant, but still both divergence and curl free. The field ##y\vec e_y - z \vec e_z## should be among the ##\ell = 2## terms.
 
  • Like
Likes Demystifier
  • #39
tom.stoer said:
The electric field must not vanish. The very first assumption is already not correct.
tom.stoer said:
but it should be clear that there's no good reason why the electric field should vanish.
Again, you can argue that the electric field vanishes by symmetry.
Orodruin said:
it is in essence a statement about the divergence of the electric field
Yes, that is how I originally posed the problem.
Orodruin said:
You can also see this in the one-dimensional analogue of the problem f′(x)=κf′(x)=κf'(x) = \kappa, which has the solutions f(x)=κx+Af(x)=κx+Af(x) = \kappa x + A with AAA being an arbitrary constant. Unless you specify the behaviour at infinity, e.g., limx→∞[f(x)+f(−x)]=2A=0limx→∞[f(x)+f(−x)]=2A=0\lim_{x\to\infty}[f(x) + f(-x)] = 2A = 0, your solution will have undetermined constants. In the 3D case, you would generally find ⃗E=ρ0⃗x/3ϵ0+⃗kE→=ρ0x→/3ϵ0+k→\vec E = \rho_0 \vec x / 3\epsilon_0 + \vec k, where ⃗kk→\vec k is a constant vector.
So you are arguing that ##\mathbf{E}\neq0##?
 
  • #40
NFuller said:
Again, you can argue that the electric field vanishes by symmetry.
No, you really cannot. To make a symmetry argument also the boundary conditions (in this case posed at infinity) must also be symmetric under whatever transformation you refer to. In the "standard" case of the point charge, you can assume the field to be vanish at infinity and this condition is perfectly symmetric under rotations. However, the case with a constant charge density is incompatible with a field that goes to zero at infinity and any behaviour at infinity is going to break your symmetry.
 
  • #41
NFuller said:
Again, you can argue that the electric field vanishes by symmetry.
No, you can't.

If you have a charge density ##\rho## and an equation which allows you to solve for ##E## in terms of ##\rho##, and if your symmetry argument results in an ##E## that does not solve the equation, then the argument violates the equation and is therefore wrong (or incompatible with the equation).

The Gauß law is a local equation. To construct a global solution you can start with a local one and extend it globally. But you must start with a solution. You must not start with some ##E## that does not solve the equation locally but hope that it does so globally.

NFuller said:
So you are arguing that ##\mathbf{E}\neq0##?
Yes.

(and if you don't like that you are free to call this situation unphysical)
 
  • Like
Likes Orodruin
  • #42
Orodruin said:
No, you really cannot. To make a symmetry argument also the boundary conditions (in this case posed at infinity) must also be symmetric under whatever transformation you refer to. In the "standard" case of the point charge, you can assume the field to be vanish at infinity and this condition is perfectly symmetric under rotations. However, the case with a constant charge density is incompatible with a field that goes to zero at infinity and any behaviour at infinity is going to break your symmetry.
So why are these same symmetry arguments used when finding the electric field due to infinite charge distributions in 1 and 2 dimensions? I have a hard time believing that ##\mathbf{E}## is non zero. If the universe looks the same in all directions out to infinity, then why would an electric field have any directional preference?
 
  • #43
tom.stoer said:
If you have a charge density ρρ\rho and an equation which allows you to solve for EEE in terms of ρρ\rho, and if your symmetry argument results in an EEE that does not solve the equation, then the argument violates the equation and is therefore wrong (or incompatible with the equation).
Or the equation is not valid in the system given. This point has already been eluded to in the paper reference by greypilgrim, where the field produced does not satisfy the field equations.
 
  • #44
Suppose the entire universe on one side of a plane has constant charge density. Then the electric field at the plane is infinite, isn't it?
Symmetry or not, this is a problem.
Now make the superposition with the charge on the other side of the plane.
The result is infinity minus infinity.
Gauss or not, something is going wrong there.
 
  • #45
NFuller said:
So why are these same symmetry arguments used when finding the electric field due to infinite charge distributions in 1 and 2 dimensions?
I assume here that you are talking about 1- and 2-dimensional charge distributions in 3-dimensional space, because the corresponding problem in 1-dimensional space does have the same problem. The point is that, unlike for the constant distribution in all of space, you can find boundary conditions at infinity that does have the same symmetries as the charge distribution itself. In particular, for the infinite line charge, the boundary condition that the field goes to zero as you go far away from the line as well as the translational symmetry along the line is satisfied by a ##\vec e_\rho/\rho## field (where ##\rho## is the radial polar coordinate) and for the infinite surface charge you can find a boundary condition such that the field goes to a constant field far away from the surface and is translationally invariant for translations within the surface. For the infinite volume charge, this is no longer possible. You cannot find a condition that is both rotationally and translationally symmetric (which are the symmetries of the charge distribution) and so you must impose boundary conditions that break these symmetries.

NFuller said:
I have a hard time believing that ##\mathbf{E}## is non zero. If the universe looks the same in all directions out to infinity, then why would an electric field have any directional preference?

The point is that the universe does not look the same in all directions and/or does not display translational invariance. You are thinking only of the charge distribution but the boundary conditions must form a part of that statement. It is simply inconsistent to assume that. You really should not have a hard time believing that ##\vec E## is non-zero. In order for ##\vec E## to be an electric field at all it must satisfy Maxwell's equations and your assertion that it is zero is a direct violation of Maxwell's equations.

Note that the derivation of the field from a point charge (for example) is directly dependent on the implicit assumption that the boundary conditions satisfy the same symmetry properties as the charge distribution. If you put different boundary conditions, you will add a divergence and curl free field to the solution and it still satisfies Maxwell's equations.
 
  • Like
Likes maline and tom.stoer
  • #46
NFuller said:
Or the equation is not valid in the system given. This point has already been eluded to in the paper reference by greypilgrim, where the field produced does not satisfy the field equations.
This is what the paper says:
One way out of it is to consider Maxwell’s two equations for electrostatics ∇ · E = ρ/ε0 , (11) and ∇ × E = 0, (12) and realize that the first one is not compatible with the answer dictated by the underlying symmetry of the distribution, that is, E = 0 everywhere is not a valid solution of Maxwell’s equations when we take ρ as a continuous uniform charge distribution fulfilling all the space.
This is not saying that Maxwell's equations are not valid. It is saying that Maxwell's equations are incompatible with the underlying symmetry of the charge distribution. The question then becomes: "Why is it incompatible?" The correct reply to that question is that it must be broken by the boundary conditions, just as indicated above, it is the only thing you have left that can break the symmetries. And not only can it break the symmetries, it has to break the symmetries when you take that charge distribution. If you are not talking about a solution to Maxwell's equations, you are really not talking about an electric field.
 
  • Like
Likes tom.stoer
  • #47
NFuller said:
Or the equation is not valid in the system given. This point has already been eluded to in the paper reference by greypilgrim, where the field produced does not satisfy the field equations.
Your intended „solution” is extremely strange; it solves a pseudo-problem, namely your (incomplete) symmetry argument, by introducing other problems.

I can‘t see any reason why this (incomplete) symmetry argument is stronger or better than well-defined solutions of Maxwell‘s equations.

A charge distribution causing problems with established physics is to be ruled out, instead of ruling out established physics.
 
  • #48
Orodruin said:
This is not saying that Maxwell's equations are not valid. It is saying that Maxwell's equations are incompatible with the underlying symmetry of the charge distribution. The question then becomes: "Why is it incompatible?" The correct reply to that question is that it must be broken by the boundary conditions, just as indicated above, it is the only thing you have left that can break the symmetries. And not only can it break the symmetries, it has to break the symmetries when you take that charge distribution. If you are not talking about a solution to Maxwell's equations, you are really not talking about an electric field.
So what boundary conditions are you assuming? What would the field look like under that assumption?
 
  • #49
NFuller said:
So what boundary conditions are you assuming? What would the field look like under that assumption?

It does not matter as long as they are compatible with the differential equation, and if they are, then they will break translational symmetry. What the field will look like has been discussed already. The general expression for the potential of the homogeneous problem in spherical coordinates is given in post #38. Regardless of the arbitrary constants, a field ##\vec E = - \nabla V## for that ##V## is going to satisfy ##\nabla \cdot \vec E = 0##. To this you add the particular solution given earlier in the same post and you will have the general solution. The arbitrary coefficients have to be fixed based on the behaviour at infinity. Note that none of the terms (except the constant ##\ell = 0## term) leads to a field that is zero at infinity, which is why you can make the assumption that the field vanishes at infinity in the case of a point charge. However, in the case of a constant charge density, you cannot cancel the growth of the particular solution in all directions simultaneously and therefore also not assume that the field vanishes at infinity.

Edit: To elaborate on this. In the same spirit as putting bounds such that the solution is zero at infinity in the case of a point charge, you can impose boundary conditions such that the field grows at most as ##r## as ##r \to \infty##. This rules out all of the modes with ##\ell \geq 1## from the expansion of the homogeneous potential and leaves the solution on the form ##\vec E = \rho \vec x/3\epsilon_0 + \vec k##.
 
  • #50
I like Serena said:
then the electric field at the plane is infinite, isn't it?
Again, this depends on you giving appropriate boundary conditions to your region of interest.
 
  • #51
Orodruin said:
Again, this depends on you giving appropriate boundary conditions to your region of interest.
Doesn't it follow directly from the principle of superposition?
Infinite force from one side (on a test charge) and no force from the other side?
 
  • #52
I like Serena said:
Doesn't it follow directly from the principle of superposition?
Sure, you can superpose solutions. But if you superpose solutions with inhomogeneous boundary conditions the superposition will satisfy boundary conditions that are the linear combination of the boundary conditions you superposed.

Also, this is not what I was replying to. The solution being infinite at the surface is not well defined. Instead, you have to impose proper boundary conditions at ##r\to \infty##. If you do so, your solution will be finite.
 
  • #53
Orodruin said:
Sure, you can superpose solutions. But if you superpose solutions with inhomogeneous boundary conditions the superposition will satisfy boundary conditions that are the linear combination of the boundary conditions you superposed.

Also, this is not what I was replying to. The solution being infinite at the surface is not well defined. Instead, you have to impose proper boundary conditions at ##r\to \infty##. If you do so, your solution will be finite.
As yet I'm not talking about solutions to Gauss's law.
@NFuller uses symmetry in an infinitely charged universe to conclude that the superposition of all forces on a test charge must be zero.
My point is that if we have a half charged universe, that same superposition would lead to an infinite force on a test charge, so a fully charged universe would lead to subtracting infinity from infinity, defeating the argument that it is supposed to be zero - the result is not-a-number (NaN), or just 'undefined'.
As such there is no contradiction to Gauss's law, since the result is undefined.
It seems to fit your conclusion that the solution to Gauss's law has an unknown constant vector (or some such) that cannot be determined.
 
Last edited:
  • #54
I like Serena said:
As yet I'm not talking about solutions to Gauss's law.
Then you are not talking about an electric field. Electric fields satisfy Gauss’s law. If you want to make your own model of electrostatics you are welcome to do so (elsewhere, personal theories are against forum rules).

I like Serena said:
@NFuller uses symmetry in an infinitely charged universe to conclude that the superposition of all forces on a test charge must be zero.
Where do you think the superposition principle comes from? It comes from the equation of motion, ie, Gauss’s law, being a linear differential equation. To use the superposition principle, you are implicitly assuming Gauss’s law. Now, it is generally not presented like this in high-school because most people have an easier time accepting heuristic arguments and ”field from separate particles”, but that does not make it any less true that superposition being possible relies on the field satisfying Gauss’s law. In the typical superposition of point charges argumentation, the second implicit assumption is that the field vanishes at infinity. This assumption can no longer hold in the case of an infinitely extended charge and in general the translational symmetry is broken by the boundary condition you have to impose.

Let me repeat the main message again, because it is important: You simply cannot ignore the symmetry (or lack thereof) of the boundary conditions if you want to apply symmetry arguments. It does not matter if the charge distribution displays a symmetry - if the boundary condition does not display the same symmetry - then it is not a symmetry of the system.

I like Serena said:
My point is that if we have a half charged universe, that same superposition would lead to an infinite force on a test charge, so a fully charged universe would lead to subtracting infinity from infinity, defeating the argument that it is supposed to be zero - the result is not-a-number (NaN), or just 'undefined'.
As such there is no contradiction to Gauss's law, since the result is undefined.
It seems to fit your conclusion that the solution to Gauss's law has an unknown constant vector (or some such) that cannot be determined.

You are making the same mistake as the OP here. You fail to account for the fact that with a charge distribution that extends to infinity, you need non-zero boundary conditions on the limiting behaviour. Undefined functions do not solve differential equations, functions with well defined behaviour do.
 
  • #55
Orodruin said:
Then you are not talking about an electric field. Electric fields satisfy Gauss’s law. If you want to make your own model of electrostatics you are welcome to do so (elsewhere, personal theories are against forum rules).

Sure, electric fields satisfy Gauss's law.
That doesn't change the fact that the superposition principle - also observed empirically - is supposed to coincide with it.
Isn't that the whole point of this thread?
 
Last edited:
  • #56
If you start from the (Lorentz-invariant) Lagrangian of electrodynamics you arrive at the Maxwell Equations if and only if charge and current vanish at infinity, else you get boundary terms. So if we consider the extremization of the action as the basic physical principal, than the answer is:

Maxwell's equations (and hence also Gauß's law) are just not valid for a constant charge density without spatial cutoff.

I think this answer is quite reasonable, since an infinitely stretched constant charge density is just unphysical. Also: If you think about how one usually derives the Gauß law in non-relativistic electrodynamics, you notice, that you always start with the integral formulation which necessarily incorporates compact volumes. The transition to the differential formulation is only valid within these volumes. The extension to whole Euclidean space can be done only if the charge density is integrable, i.e. if there exists a finite total charge.

You could also argue, that a constant charge density on the whole space is equivalent to empty space with shifted ground state energy. Since except for GR the total energy is meaningless in physics (only energy differences matter), constant charge density (infinitely stretched) and empty space are totally equivalent in non-gravitational electrodynamics, if you neglect possible further quantum influences. Hence the solution is trivially a constant E-field.
 
Last edited by a moderator:
  • #57
Orodruin said:
It does not matter as long as they are compatible with the differential equation
The solution doesn't set the boundary condition, the boundary condition sets the solution. Even with some boundary condition at infinity, how do you know where to set the origin in space to make the electric field you suggested valid? Infinity is the same distance away from every point in space, so the boundary condition doesn't really set an origin.

I have been looking into some more generalized constructions of Maxwell's equations. In particular, that the action becomes zero if the fields satisfy the correct equations of motion, which are of course the Maxwell Equations. However, the Lagrangian formalism again relies on the fields decaying at infinity. I'm wondering if the principle of least action would lead to a different solution, or any solution, without assuming the fields vanish at infinity.
 
  • #58
NFuller said:
The solution doesn't set the boundary condition, the boundary condition sets the solution.
I never said that. I said that you have to put boundary conditions that are compatible with the differential equation.

NFuller said:
Even with some boundary condition at infinity, how do you know where to set the origin in space to make the electric field you suggested valid? Infinity is the same distance away from every point in space, so the boundary condition doesn't really set an origin.
That is not how infinity works. Look at the (spatially) one-dimensional case ##E' = \kappa##, where you obtain ##E = \kappa x + A##. A behaviour at infinity that fixes ##A## will be of the type ##\lim_{x\to \infty} [E(x) + E(-x)] = E_0## and set ##A = E_0/2## and therefore single out ##x = - E_0/2\kappa## as the point with ##E = 0##. Your constructions in the three-dimensional case will look similar and instead of the sum of two points involve integrals over the spherical harmonics.

NFuller said:
In particular, that the action becomes zero if the fields satisfy the correct equations of motion, which are of course the Maxwell Equations.
Yes, the action may be zero, but that is not what you want to find when you do Lagrangian mechanics. You want to find out for which field configurations the variation of the action is zero, i.e., when the action is stationary.

Edit: Also, another pet peeve of mine. "Principle of least action" is a confusing misnomer, it should really be called the "principle of stationary action".

Metmann said:
If you start from the (Lorentz-invariant) Lagrangian of electrodynamics you arrive at the Maxwell Equations if and only if charge and current vanish at infinity, else you get boundary terms.
The action would formally be infinite, but that does not really stop you from having variations of the fields that are localised, which will give you finite variations without boundary terms and those variations will give you Maxwell's equations. Even if you allow variations that do not vanish at infinity, the variation of the action must be zero also for those variations that do vanish. This is the underlying reason why, when dealing with variations on spaces with a boundary, the Euler-Lagrange equations always hold and the effect of allowing variations on the boundary is to impose natural boundary conditions. A good question might be what those natural boundary conditions would be in this scenario. I suspect that you will get inconsistencies there.

Metmann said:
If you think about how one usually derives the Gauß law in non-relativistic electrodynamics, you notice, that you always start with the integral formulation which necessarily incorporates compact volumes. The transition to the differential formulation is only valid within these volumes. The extension to whole Euclidean space can be done only if the charge density is integrable, i.e. if there exists a finite total charge.
You do not need the integral form of Gauss's law to hold for the entire space to derive the infinitesimal form. You only need it to hold for any compact sub-volume. In this context I would also point out that fundamental laws are not derived, but introduced based on observation. The "derivation" of Gauss's law in electrodynamics is essentially a collection of arguments starting from observed properties of electromagnetism that just so happen to be the properties of solutions to the Laplace equation. In fact, this was actually already set in stone once you observed and assumed the ##1/r^2## behaviour of the potential of a point charge and the superposition of fields, as the Green's function of the Laplace equation just goes as ##1/r^2## and the superposition principle requires a linear differential equation.

Metmann said:
You could also argue, that a constant charge density on the whole space is equivalent to empty space with shifted ground state energy.
To be honest, this sounds very fishy and ad hoc. Can you back up how it would be equivalent to empty space?
 
Last edited:
  • Like
Likes tom.stoer
  • #59
You may want to study this equation in one spatial dimension. There are essentially four different topologies: ##R## = the real line, ##R^+## which allows you to add a static charge at ##x = 0##, ##[x,L]## which allows for two charges, and compact ##S^1## w/o boundary. You may derive the Maxwell equations including boundary terms and investigate whether a topology with some appropriate boundary condition allows for constant ##\rho## and vanishing ##E = 0##.

For ##S^1## constant charge always implies ##\rho = 0##.
 
Last edited:
  • Like
Likes Orodruin
  • #60
Orodruin said:
when dealing with variations on spaces with a boundary, the Euler-Lagrange equations always hold and the effect of allowing variations on the boundary is to impose natural boundary conditions. A good question might be what those natural boundary conditions would be in this scenario. I suspect that you will get inconsistencies there.

That's what I mean with 'boundary terms'. I also don't think there is a natural and consistent way to choose these.

Orodruin said:
The "derivation" of Gauss's law in electrodynamics is essentially a collection of arguments starting from observed properties of electromagnetism that just so happen to be the properties of solutions to the Laplace equation.

You can also 'derive' it using the classical concept of charge conservation and that uses the integral formulation of Gauß's law.

Orodruin said:
In fact, this was actually already set in stone once you observed and assumed the 1/r21/r^2 behaviour of the potential of a point charge and the superposition of fields, as the Green's function of the Laplace equation just goes as 1/r21/r^2 and the superposition principle requires a linear differential equation.

That's indeed true. I am curious: Does this 'derivation' break down somewhere during the transition from the local statement (localized point charge) to the global statement (uncountably infinite number of point charges spread to infinity)?

Orodruin said:
To be honest, this sounds very fishy and ad hoc. Can you back up how it would be equivalent to empty space?

If you consider just electrostatics, a constant infinitely spread charge density ##\rho## amounts to an infinite total energy
$$ E \sim \rho^2 \int_{\mathbb{R}^3} \frac{\mathrm{d}^3 \vec{x} \, \mathrm{d}^3\vec{x}'}{|\vec{x}-\vec{x}'|} = \infty. $$
Energy density would be
$$ e(\vec{x}) \sim \rho^2 \int_{\mathbb{R}^3} \frac{\mathrm{d}^3\vec{x}'}{|\vec{x}-\vec{x}'|}, $$
which is also infinite everywhere, but the integrand is definitely translationally invariant, hence energy density is of course homogeneous.
Since there is no difference between any pair of points, I would suggest to treat the infinite ##E## as a vacuum energy, hence subtracting it from the physical energy by setting ##\rho=0##.
In my opinion, only spatial and temporal deviations of physical quantities have physical meaning in classical physics. Hence, the subtraction of the infinite background energy does not change the physics.
Of course here I neglected possible varying mass density or some temporal behaviour.

tom.stoer said:
For S1S^1 constant charge always implies ρ=0\rho = 0.
Interesting. What about ##S^n## in general? My first guess would be, that for ##n>=2## the situation is different due to simply connectedness. Solving the problem for ##S^3## instead of ##\mathbb{R}^3## would probably help.
 
Last edited by a moderator:
  • #61
This question is actually of significant historical importance, but in the context of Newton's gravity, rather than Maxwell's equations. The basic issue for static electric fields is of course the same.

Newton assumed the universe should be infinite and homogeneous, and he came to the same resolution as NFuller, that the gravitational field should be zero by symmetry arguments-- if it doesn't know how to point, it must be zero. He regarded that as a deeper principle than the equations he was solving, so although the issues about the problems with applying symmetry arguments to Gauss' law that have been raised are entirely valid, Newton, who knew the necessary version of Gauss' law (i.e. that a spherically symmetric mass distribution had to produce a field toward the origin that depended only on the enclosed charge), felt the symmetry argument supercedes the mathematics of forming a proper solution. Perhaps it could be said he felt the boundary condition must be whatever is necessary to produce a zero field, because a field that doesn't know which way to point must be zero. The point is, Newton felt his gravity was consistent with a static universe, and this mistake caused him to miss that the universe should be dynamical.

Of course general relativity reaches a different conclusion about an infinite homogeneous mass distribution, but that's a different theory, so it's easy to overlook that Newton could also have realized the universe should be dynamical. I have always felt that Newton was basically wrong, in the sense that even if Newton's own theory of gravity were applicable, the gravitational field should not be taken to be zero, and the universe could not be static. I think the problem is in the implicit assumption that the field must be a "thing," handed to us without our input or participation, so must be unique. But this is never actually required in empirical sciences-- we only need the observations to come out as predicted, including whatever input or participation we need to supply. The observation is that a homogeneous mass distribution that is bounded and stationary undergoes contraction, and the timescale for density increase is given by 1/sqrt(rho*G). A similar expression would hold for the expansion of a homogeneous charge distribution with fixed charge-to-mass ratio. Notice this timescale depends on the local density, so the global scale is irrelevant and should exhibit no problems even when extended to infinity. The contraction/expansion timescale is what must be unique, not the fields, because observing fields requires a somewhat arbitrary calibration procedure, which may need to dovetail with the frame of reference or perspective of the observer.

So I think the physical problem here is in the implicit assumption that the field must be unique. It seems odd that the field you calculate could depend on where you set the origin, but that's not odd if the behavior you are trying to understand (contraction or expansion toward or away from some origin) also depends on where you set the origin-- the timescale for the contraction or expansion does not depend on where you set the origin. Also, the OP was bothered by the increasing magnitude of the field as you go away from the origin, but this can depend on the calibration procedure you use to measure fields, and there is no rule that says the field cannot depend on the calibration, nor that it cannot grow without limit if that's how you choose to calibrate it. All that has to be satisfied in an empirical science is that once you choose your origin (i.e., local reference frame), and once you have a self-consistent procedure for obtaining correct results to all your measurements from that perspective, then all observers must get the results they predicted, even if those results depend on the procedure used.

In short, I feel we should never have assumed the field must be unique, as that assumption is more than we get to assume when using empirical science. I agree with Newton that the physical outcome should always trump the formal mathematical issues, but I would add that a physical outcome is a description by an observer using a given procedure, and is not in and of itself something that needs to be unique. This is very much the perspective of relativity, so I feel that relativity has already solved the issue even without replacing Newton's gravity with Einstein's, which is reasonable because the solution can hold even at times when the speeds are much less than the speed of light. Hence I'm saying not only that I agree the solution is allowed to depend on implicit boundary conditions, but also that different boundary conditions are allowed within the same physical problem, if they dovetail with different procedures by different observers. Relativity basically says that it is only necessary that the observers be right, not that they agree!
 
Last edited:
  • #62
NFuller said:
If we consider a uniform charge density ρρ\rho extending through all space
Other people have basically mentioned this, but the divergence theorem (and Stokes' theorem in general) only holds when ##V## is a compact subset of ##\mathbb{R}^n##. ##\mathbb{R}^n## itself is not compact. Thus you aren't guaranteed that
$$\int_{\mathbb{R}^n} \nabla\cdot{\bf F}dV=\int_{\partial \mathbb{R}^n} {\bf F}\cdot {\bf dA}$$
So you can't make the claim that ##\nabla\cdot\bf E=\rho/\varepsilon_0##. I'm not even clear if it's possible to formulate the meaning of ##\partial \mathbb{R}^n## in a coherent way.
 
Last edited:
  • Like
Likes NFuller
  • #63
Ken G said:
So I think the physical problem here is in the implicit assumption that the field must be unique ...

... and is not in and of itself something that needs to be unique. This is very much the perspective of relativity, so I feel that relativity has already solved the issue
Field equations for the electric field do have unique solutions if boundary conditions are included.

Ken G said:
Hence I'm saying not only that I agree the solution is allowed to depend on implicit boundary conditions, but also that different boundary conditions are allowed within the same physical problem
Changing the boundary or boundary condition usually changes the physical problem.
 
  • #64
tom.stoer said:
Changing the boundary or boundary condition usually changes the physical problem.
Which is my point-- it is appropriate to change the physical problem when one changes the observer's perspective/procedure. So it's not a mathematics problem, or a symmetry problem. It's a physics problem, so requires a physics solution, by which I mean, a solution that merely connects the observer to their own predictions, with no need to use an absolute language about what is happening.
 
  • #65
Ok, sorry, then somehow I got you wrong
 
  • #66
There could be an important difference between Newton's gravity and Coulomb's electric force, in that all matter has the same gravitational charge-to-mass ratio, whereas we can build instruments with various different charge-to-mass ratios. Hence, one does not say that the Coulomb force admits to an equivalence principle. Still, that issue doesn't seem to be the crux of the problem with using Gauss' law in the gravitational context, so it seems a similar solution as relativity finds in the gravitational case should be possible in the Coulomb case, even in the limit of taking c to infinity so no new theory would be needed. We would only need to relax the requirement that physics regard itself as enabled to make absolute statements like "what is the actual field here."

ETA: Let me express this point in terms of a trivial but interesting theorem that applies to any sphere of constant charge density. Let the origin be at the center, and then the usual electric field is equal to -x, where x is the displacement vector from the origin (and the charge density is suitably scaled). Similarly, the field at y is -y. But now let us regard y as a new origin for our coordinates, such that the coordinates at x become x-y. If an observer at y simply adds y to every field we calculated before, then the field at x is -x+y, which again obeys the same rule-- it is negative the displacement from the new origin! What this means is, if we have a spherical homogeneous charge distribution, we can regard any point within it as the origin where the field is zero, and use Gauss' law around that point, ignoring all charges outside the Gaussian sphere, as long as we are willing to have a field that differs from the original by a fixed vector y. All we then need is a way to regard that fixed field difference as an ignorable difference (stemming from the different perspectives of the observers), and we allow all observers to correctly infer what they will observe, without any need for a concept of an "absolute field," or any machinery other than the integral form of Gauss' law.

Notice this is the 3D homogeneous analog to the points made above in 1D involving an arbitrary constant of integration, but my point is that this constant of integration is not a different boundary condition in the sense of a different physical situation, it is a different constraint reflecting a different point of reference within the same physical situation. Hence we should not say the nonuniqueness of the field stems from insufficiently describing the physical situation, we should say it is nonunique because we have insufficiently described the point of reference of the observer, and what aspects of that observer's reality that we are allowed to regard as ignorable differences between observers. That solves Newton's problem and gives rise to the equivalence principle, but for electric forces, the problem seems perhaps trickier because an equivalence principle may not exist. (Yet people do seek ways to unify gravity with electric forces, so perhaps it is indeed a similar situation)
 
Last edited:
  • #67
Orodruin said:
I said that you have to put boundary conditions that are compatible with the differential equation.
This is what is not making sense to me. Generally speaking, you can set the boundary condition to whatever you want: a constant, a sine function, a spherical harmonic, etc. I don't need to pick a boundary condition that makes a particular solution work. I can pick anything, and the solution must conform to that boundary condition.
Orodruin said:
That is not how infinity works. Look at the (spatially) one-dimensional case E′=κE′=κE' = \kappa, where you obtain E=κx+AE=κx+AE = \kappa x + A. A behaviour at infinity that fixes AAA will be of the type limx→∞[E(x)+E(−x)]=E0limx→∞[E(x)+E(−x)]=E0\lim_{x\to \infty} [E(x) + E(-x)] = E_0 and set A=E0/2A=E0/2A = E_0/2 and therefore single out x=−E0/2κx=−E0/2κx = - E_0/2\kappa as the point with E=0E=0E = 0. Your constructions in the three-dimensional case will look similar and instead of the sum of two points involve integrals over the spherical harmonics.
Again, it is not clear how the origin is set here. Am I at the origin, or is two meters to my right the origin? I see that you have found where ##E=0## with respect to the origin, but you have not defined where that origin actually is.
 
  • #68
TeethWhitener said:
Other people have basically mentioned this, but the divergence theorem (and Stokes' theorem in general) only holds when ##V## is a compact subset of ##\mathbb{R}^n##. ##\mathbb{R}^n## itself is not compact. Thus you aren't guaranteed that
$$\int_{\mathbb{R}^n} \nabla\cdot{\bf F}dV=\int_{\partial \mathbb{R}^n} {\bf F}\cdot {\bf dA}$$
So you can't make the claim that ##\nabla\cdot\bf E=\rho/\varepsilon_0##. I'm not even clear if it's possible to formulate the meaning of ##\partial \mathbb{R}^n## in a coherent way.
I've read about this before but did not really understand it. Could you explain more about what a compact subset is? I'm not very familiar with this terminology.
 
  • #69
A compact subset in ##\mathbb{R}^n## is a closed and limited subset, meaning it has a boundary and is 'finite' in the sense that it does not stretch out to infinity. There is a more general topological definition, but that's not the matter here.
The divergence theorem holds in general only for compactly supported ##\nabla \cdot \mathbf{F}## or equivalently only on compact subsets of manifolds, because this (plus continuity) ensures existence of the integral. ##\mathbb{R}^n## does not have a natural boundary a priori.(and of course it is not compact, because it stretches out to infinity).
 
  • Like
Likes NFuller
  • #70
Metmann said:
That's indeed true. I am curious: Does this 'derivation' break down somewhere during the transition from the local statement (localized point charge) to the global statement (uncountably infinite number of point charges spread to infinity)?
The 1/r^2 Green's function pre-assumes that the field of the point charge vanishes at infinity. The integrals you will obtain from this will clearly not be convergent (working in the potential now, which is usually much more convenient than working with the fields). This is just the point you noticed about the total potential energy.
NFuller said:
Generally speaking, you can set the boundary condition to whatever you want
This is not generally true. Take f’(x) = 1/x. Having a boundary condition f(0) = 1 is clearly incompatible with this differential equation. Just an example.

NFuller said:
Again, it is not clear how the origin is set here.
The origin is arbitrary. The natural choice would be the point where E=0. If not you will end up with a different boundary condition. The boundary condition is not invariant under translations.

TeethWhitener said:
Other people have basically mentioned this, but the divergence theorem (and Stokes' theorem in general) only holds when ##V## is a compact subset of ##\mathbb{R}^n##. ##\mathbb{R}^n## itself is not compact. Thus you aren't guaranteed that
$$\int_{\mathbb{R}^n} \nabla\cdot{\bf F}dV=\int_{\partial \mathbb{R}^n} {\bf F}\cdot {\bf dA}$$
So you can't make the claim that ##\nabla\cdot\bf E=\rho/\varepsilon_0##. I'm not even clear if it's possible to formulate the meaning of ##\partial \mathbb{R}^n## in a coherent way.
As already stated, this is not what is done when going from the integral form of Gauss’s law to the differential form. You only need the divergence theorem for compact volumes to do that.
 

Similar threads

Replies
11
Views
5K
  • Introductory Physics Homework Help
Replies
1
Views
820
  • Introductory Physics Homework Help
Replies
17
Views
400
Replies
7
Views
1K
  • Introductory Physics Homework Help
Replies
12
Views
4K
  • Classical Physics
Replies
5
Views
2K
  • Other Physics Topics
Replies
5
Views
3K
  • Classical Physics
Replies
12
Views
2K
  • Introductory Physics Homework Help
Replies
9
Views
2K
  • Introductory Physics Homework Help
Replies
12
Views
2K
Back
Top